www.vorhilfe.de
Vorhilfe

Kostenlose Kommunikationsplattform für gegenseitige Hilfestellungen.
Hallo Gast!einloggen | registrieren ]
Startseite · Forum · Wissen · Kurse · Mitglieder · Team · Impressum
Forenbaum
^ Forenbaum
Status Englisch
  Status Grammatik
  Status Lektüre
  Status Korrekturlesen
  Status Übersetzung
  Status Sonstiges (Englisch)

Gezeigt werden alle Foren bis zur Tiefe 2

Navigation
 Startseite...
 Neuerdings beta neu
 Forum...
 vorwissen...
 vorkurse...
 Werkzeuge...
 Nachhilfevermittlung beta...
 Online-Spiele beta
 Suchen
 Verein...
 Impressum
Das Projekt
Server und Internetanbindung werden durch Spenden finanziert.
Organisiert wird das Projekt von unserem Koordinatorenteam.
Hunderte Mitglieder helfen ehrenamtlich in unseren moderierten Foren.
Anbieter der Seite ist der gemeinnützige Verein "Vorhilfe.de e.V.".
Partnerseiten
Weitere Fächer:

Open Source FunktionenplotterFunkyPlot: Kostenloser und quelloffener Funktionenplotter für Linux und andere Betriebssysteme
Forum "Reelle Analysis mehrerer Veränderlichen" - Rotationsinvarianz Laplace
Rotationsinvarianz Laplace < mehrere Veränderl. < reell < Analysis < Hochschule < Mathe < Vorhilfe
Ansicht: [ geschachtelt ] | ^ Forum "Reelle Analysis mehrerer Veränderlichen"  | ^^ Alle Foren  | ^ Forenbaum  | Materialien

Rotationsinvarianz Laplace: Verständnisfrage
Status: (Frage) überfällig Status 
Datum: 22:05 Sa 17.05.2014
Autor: SiuNimTau

Aufgabe
Es gibt allerdings auch eine kurze, koordinatenfreie Herleitung für die O(n)-Invarianz des Laplace-Operator:
Sie benutzt, dass man [mm] \Delta [/mm] = div ° grad schreiben kann, wobei div und grad analog wie in [mm] \IR^3 [/mm] definiert sind.
Bezeichnet Df die Jakobi-Matrix von f, so ist für [mm] u:\IR^n [/mm] -> [mm] \IR [/mm]
grad u = [mm] (Du)^T [/mm]
bezüglich des Standardskalarproduktes (klar, wenn man die Komponenten von Du mit denen von grad u vergleicht).
Für ein F: [mm] \IR^n [/mm] -> [mm] \IR^n [/mm] ist dann
div F = tr(DF)
wobei tr die Spur, also die Summe über den Diagonalelementen von F bezeichnet.
Somit gilt:
[mm] \Delta [/mm] u = div grad u = [mm] tr(D(Du)^T) [/mm]
Ist nun v = u ° A wie oben, so gilt
Dv(x) = Du(Ax) ° A(x) => grad v(x) = [mm] A^T [/mm] grad u(Ax)
Nun erhält man
div grad v(x) = [mm] tr(D(A^T [/mm] grad u(Ax))) = [mm] tr(A^T [/mm] D(grad u)(Ax) A) = tr(A^(-1) D(grad u)(Ax) A) = tr(D(grad u)(Ax)) = div grad u (Ax) = [mm] \Delta [/mm] u (Ax)

Hallo zusammen,

das hier hab ich als Beweis für die Rotationsinvarianz des Laplace Operators gefunden.

Ich verstehe eigentlich alles, außer bei der Beweisführung am Ende das 2. Gleichheitszeichen. Das man [mm] $A^T$ [/mm] rausziehen kann, leuchtet mir ein und auch das das (Ax), aber woher kommt plötzlich das A??

Würde mich freuen, wenn mir jemand hilft, das zu verstehen.


        
Bezug
Rotationsinvarianz Laplace: Fälligkeit abgelaufen
Status: (Mitteilung) Reaktion unnötig Status 
Datum: 22:20 Mo 19.05.2014
Autor: matux

$MATUXTEXT(ueberfaellige_frage)
Bezug
Ansicht: [ geschachtelt ] | ^ Forum "Reelle Analysis mehrerer Veränderlichen"  | ^^ Alle Foren  | ^ Forenbaum  | Materialien


^ Seitenanfang ^
www.englischraum.de
[ Startseite | Forum | Wissen | Kurse | Mitglieder | Team | Impressum ]